2
$\begingroup$

Let $L/K$ be a field extension. Suppose $f(x)$ is the minimal polynomial of $\alpha$ and $\alpha'$ over $K$ and suppose $g(x)$ is the minimal polynomial of $\beta$ and $\beta'$ over $K$. If $L = (\alpha, \beta)$, is it true that the rules $\sigma(\alpha) = \alpha'$ and $\sigma(\beta) = \beta'$ define a $K$-automorphism of $L$?

For example: $\mathbb{Q} \subset \mathbb{Q}(\sqrt{7}, \sqrt{13})$. I wish to define an automorphism $\sigma$ satisfying $\sigma(\sqrt{7}) = - \sqrt{7}$ and $\sigma(\sqrt{13}) = \sqrt{13}$. That is, I put $\sigma(a + b \sqrt{7} + c\sqrt{13} + d \sqrt{91}) = a - b \sqrt{7} + c \sqrt{13} - d \sqrt{91}$. Will this always extend to a ring morphism?

If this is not true, then how do I define an automorphism in practice? Do I always have to check that $\sigma$ is multiplicative?

$\endgroup$
1
  • $\begingroup$ I think it isn't always possible to define an automorphism in this manner, because the roots of $g$ and $f$ could be algebraically dependent, which would "force" the automorphism to map $\beta$ to a certain element if we set $\sigma(\alpha)=\alpha'$ $\endgroup$ Commented Nov 16 at 21:40

1 Answer 1

2
$\begingroup$

This is not possible, take $\alpha'=\alpha$, $\beta=\alpha$ and $\beta'\neq\beta$, such an automorphism $\sigma$ should satisfy the identities $\sigma(\alpha)=\alpha'=\alpha$ and $\sigma(\alpha)=\sigma(\beta)=\beta'$ so that $\alpha=\beta'$, which is not the case.

Maybe this answer won't suit you, so let's suppose $\alpha\neq\beta$. This is still not possible in general. Take $\alpha=\sqrt[3]{2},\alpha'=j\sqrt[3]{2},\beta=j,\beta'=j^2$ and $K=\mathbb{Q}$, in this case ${\rm Gal}(L/K)\simeq\mathfrak{S}_3$. There are $2$ types of automorphisms of $L/K$ :

  • the $3$-cycles, from which only $\sqrt[3]{2}\mapsto j\sqrt[3]{2}\mapsto j^2\sqrt[3]{2}\mapsto\sqrt[3]{2}$ sends $\alpha$ to $\alpha'$, but in this case $\sigma(j)=j$,
  • the transpositions, but these fix $\mathbb{Q}(\alpha)$ so we can't have $\sigma(\alpha)=\alpha'$,

so there is no such $\sigma$.

$\endgroup$
2
  • $\begingroup$ Nice! So I guess the best way to approach this is to use primitive element theorem? It seems quite impractical to check $\sigma(ab) = \sigma(a) \sigma(\beta)$ every time. $\endgroup$ Commented Nov 16 at 23:30
  • 1
    $\begingroup$ The primitive element theorem works, but in practice it is hard to find a primitive element (though it is VERY useful in theory). You can also use inductive arguments, for instance if $M/K$ and $L/K$ are both Galois, the restriction map $$\sigma\in{\rm Gal}(M/K)\mapsto\sigma_{|L}\in{\rm Gal}(L/K)$$ is surjective. $\endgroup$ Commented Nov 16 at 23:52

You must log in to answer this question.

Start asking to get answers

Find the answer to your question by asking.

Ask question

Explore related questions

See similar questions with these tags.